Erklären Sie, wie dieses Integral hergeleitet wird, indem Sie zu Polarkoordinaten wechseln.

Ich habe mir den folgenden Beitrag im Physik-Stack-Austausch über das Coulomb-Feld angesehen

https://physics.stackexchange.com/questions/7462/fourier-transform-of-the-coulomb-potential?newreg=edecf6cd6c9a404ba842fd7dfb093be8

In Pablos Antwort hat er das Integral

R 3 1 ( 2 π ) 3 e ich k R k 2 + A 2 D 3 k

Was es nach dem Umschalten auf Polarkoordinaten wird

1 ( 2 π ) 3 0 k 2 k 2 + A 2 D k 0 2 π D ϕ π / 2 π / 2 cos θ e ich k R Sünde θ D θ

Ich kann jedoch nicht verstehen, wie dies abgeleitet wurde, denn wenn k = ( k 1 , k 2 , k 3 ) Und R = ( ( R 1 , R 2 , R 3 ) Kugelkoordinaten sind

k 1 = ρ cos θ Sünde ϕ
k 2 = ρ Sünde θ Sünde ϕ
k 3 = ρ cos ϕ
So
e ich k R
würde werden
e ich ( ρ cos θ Sünde ϕ R 1 + ρ Sünde θ Sünde ϕ R 2 + ρ cos ϕ R 3 )
Ich kann nicht sehen, wie dies dem angegebenen Ergebnis ähnelt. Können Sie erklären, wie dies abgeleitet wird?

Nur ein Hinweis für den eiligen Leser - die ϕ der Polarkoordinaten in der Physik ist die θ der Polarkoordinaten in der Mathematik....
Ihre Formel für sphärische Koordinaten ist falsch; Abgesehen davon verwenden Sie die Koordinaten der Mathematiker k 3 ist falsch, aber die Integrationsformel verwendet die Koordinaten der Physiker, die auf Maß umgestellt sind θ von dem X j -Flugzeug eher als von der z -Achse. Was für ein Chaos!
Ergänzend zu Teds Kommentar und jetzt zu Marks Antwort unten: Zunächst einmal ist das anfängliche Integral rotationsinvariant - es hängt nur von der Größe ab R von R . Davon kann man also ausgehen R liegt auf der z Achse. Ich denke, das wollte Pablo schreiben - dh er hat einen Tippfehler - als er direkt vor seiner Berechnung schrieb: "Wenn wir Polarkoordinaten mit verwenden k in der z-Achse haben wir ...“ Immerhin integriert er aus/über k . ...
... Zweitens: Wenn θ ist der Winkel über/von der X j Flugzeug [das scheint nicht so zu sein, wie du es zu nehmen scheinst, aber wie Pablo zu sein scheint], das haben wir auf jeden Fall k 3 = k Sünde θ , Und R k = R k Sünde θ .
@guin_go Bitte lassen Sie mich wissen, wie ich meine Antwort verbessern kann. Ich möchte Ihnen wirklich die bestmögliche Antwort geben. –

Antworten (1)

Lassen F ( R ) durch dieses Integral gegeben sein

F ( R ) = 1 ( 2 π ) 3 R 3 e ich k R k 2 + A 2 D 3 k

Mit festem R , wenden wir eine Rotationstransformation in an k -Raum (dh, k k ' ) anpassen k z ' mit R .

Beachten Sie, dass k 2 = k ' 2 Und k R = k ' R cos ( θ ' ) . Dann ist insofern der Jacobi einer Rotationstransformation gleich 1 , das behaupten wir

F ( R ) = 1 ( 2 π ) 3 R 3 e ich k R k 2 + A 2 D 3 k = 1 ( 2 π ) 3 R 3 e ich k ' R cos ( θ ' ) k ' 2 + A 2 D 3 k ' = 1 ( 2 π ) 3 0 2 π 0 π 0 R 3 e ich k ' R cos ( θ ' ) k ' 2 + A 2 k ' 2 Sünde ( θ ' ) D k ' D θ ' D ϕ '

Kannst du jetzt fertig werden?

@guin_go Bitte lassen Sie mich wissen, wie ich meine Antwort verbessern kann. Ich möchte Ihnen wirklich die bestmögliche Antwort geben.
@guin_go Und fühlen Sie sich frei, nach oben zu stimmen und eine Antwort zu akzeptieren, wie Sie es für richtig halten. ;-)